ACLS Certification
Practice Test

Question 3 of 3
A 35-year old woman has palpitations, lightheadedness, and a stable tachycardia. The monitor shows a regular narrow-complex QRS at a rate of 180/min. Vagal maneuvers have not been effective in terminating the rhythm. An IV has been established. What drug should be administered IV?
Adenosine 6 mg
Lidocaine 1 mg/kg
Epinephrine 2-10 ụg/kg per minute
Atropine 0.5 mg